Polynomial 1,2,3.

Algebra Level 4

Let f ( x ) = x 4 + a x 3 + b x 2 + c x + d f\left( x \right) ={ x }^{ 4 }+a{ x }^{ 3 }+b{ x }^{ 2 }+cx+d , for that a , b , c , d a,b,c,d are constants. If f ( 1 ) = 1993 , f ( 2 ) = 3986 , f ( 3 ) = 5979 f\left( 1 \right) =1993,\quad f(2)=3986,\quad f(3)=5979 . Calculate the value of 1 4 [ f ( 11 ) + f ( 7 ) ] \frac { 1 }{ 4 } [f(11)+f(-7)] .


The answer is 5233.

This section requires Javascript.
You are seeing this because something didn't load right. We suggest you, (a) try refreshing the page, (b) enabling javascript if it is disabled on your browser and, finally, (c) loading the non-javascript version of this page . We're sorry about the hassle.

4 solutions

We note that:

{ f ( 1 ) = 1993 = 1 × 1933 f ( 2 ) = 3986 = 2 × 1993 f ( 3 ) = 5979 = 3 × 1993 \begin {cases} f(1) = 1993 = 1\times 1933 \\ f(2) = 3986 = 2\times 1993 \\ f(3) = 5979 = 3\times 1993 \end {cases}

So we can induce that:

f ( x ) = ( x 1 ) ( x 2 ) ( x 3 ) ( x + α ) + 1933 x f(x) = (x-1)(x-2)(x-3)(x + \alpha) + 1933x

{ f ( 11 ) = ( 10 ) ( 9 ) ( 8 ) ( 11 + α ) + 1933 ( 11 ) = 29843 + 720 α f ( 7 ) = ( 8 ) ( 9 ) ( 10 ) ( 7 + α ) + 1933 ( 7 ) = 8911 720 α \Rightarrow \begin {cases} f(11) = (10)(9)(8)(11 + \alpha) + 1933(11) & = 29843 + 720\alpha \\ f(-7) = (-8)(-9)(-10)(-7 + \alpha) + 1933(-7) & = -8911 - 720\alpha \end {cases}

1 4 [ f ( 11 ) + f ( 7 ) ] = 1 4 ( 29843 + 720 α 8911 720 α ) \Rightarrow \frac {1}{4} [f(11)+f(-7)] = \frac {1}{4} (29843 + 720\alpha -8911 - 720\alpha)

= 1 4 ( 20932 ) = 5233 \quad \quad \quad \quad \quad \quad \quad \quad \quad \space = \frac {1}{4} (20932) = \boxed {5233}

Nice solution!

Ellen Shang - 6 years, 6 months ago

Hi, can someone please explain where does the ( x 1 ) ( x 2 ) ( x 3 ) ( x + α ) + 1993 x (x-1)(x-2)(x-3)(x+\alpha)+1993x come from?

Pedro Varela - 6 years, 6 months ago

Log in to reply

Since f ( x ) = 1993 x f(x) = 1993x for x = 1 , 2 , 3 x = 1,2,3 . We can think of f ( x ) f(x) as f ( x ) = ( x 1 ) ( x 2 ) ( x 3 ) g ( x ) + 1993 x f(x) = (x-1)(x-2)(x-3)g(x) + 1993x . Notice that ( x 1 ) ( x 2 ) ( x 3 ) g ( x ) = 0 (x-1)(x-2)(x-3)g(x) = 0 , when x = 1 , 2 , 3 x = 1,2,3 , therefore leaving f ( x ) = 1993 x f(x) = 1993x , when x = 1 , 2 , 3 x = 1,2,3 . Since f ( x ) f(x) is a 4 t h 4^{th} degree polynomial whose highest power of x x is 4 4 , g ( x ) g(x) must be of the 1 s t 1^{st} degree, because ( x 1 ) ( x 2 ) ( x 3 ) (x-1)(x-2)(x-3) is of the 3 r d 3^{rd} degree. That is g ( x ) g(x) is of the form g ( x ) = β x + α g(x) = \beta x + \alpha . And since f ( x ) f(x) is a monolithic polynomial. that is the coefficient of x x of the highest power ( x 4 x^4 ) is 1 1 . Therefore, β \beta must be 1 1 , this implies g ( x ) = x + α g(x) = x + \alpha .

f ( x ) = ( x 1 ) ( x 2 ) ( x 3 ) ( x + α ) + 1993 x f(x) = (x-1)(x-2)(x-3)(x+\alpha) + 1993x

= x 4 + ( 1 + 2 + 3 + α ) x 3 + ( 1 ˙ 2 + 2 ˙ 3 + 3 ˙ 1 + 1 ˙ α + 2 ˙ α + 3 ˙ α ) x 2 \quad \quad = x^4 + (1+2+3+\alpha)x^3 + (1\dot{}2+2\dot{}3+3\dot{}1+1\dot{}\alpha+2\dot{}\alpha+3\dot{}\alpha)x^2 + ( 1 ˙ 2 ˙ 3 + 1 ˙ 2 ˙ α + 1 ˙ 3 ˙ α + 2 ˙ 3 ˙ α + 1993 ) x + 1 ˙ 2 ˙ 3 ˙ α \quad \quad \quad + (1\dot{}2\dot{}3 + 1\dot{}2\dot{}\alpha + 1\dot{}3\dot{}\alpha + 2\dot{}3\dot{}\alpha + 1993)x + 1\dot{}2\dot{}3\dot{}\alpha

= x 4 + ( 6 + α ) x 3 + ( 11 + 6 α ) x 2 + ( 1999 + 11 α ) x + 6 α \quad \quad = x^4 + (6+\alpha)x^3 + (11+6\alpha)x^2 + (1999 + 11\alpha )x + 6\alpha

Please note that f ( x ) = x 4 + ( 6 + α ) x 3 + ( 11 + 6 α ) x 2 + ( 1999 + 11 α ) x + 6 α f(x) = x^4 + (6+\alpha)x^3 + (11+6\alpha)x^2 + (1999 + 11\alpha )x + 6\alpha is of the form: f ( x ) = x 4 + a x 3 + b x 2 + c x + d f(x) = x^4+ ax^3 + bx^2 + cx + d .

Pedro, you may want to practice more on polynomial on brilliant.org, such as: https://brilliant.org/practice/factor-polynomials-higher-degress/?subtopic=polynomials&chapter=polynomial-factoring

Chew-Seong Cheong - 6 years, 6 months ago
Cody Johnson
Dec 8, 2014

Let g ( x ) = f ( x ) x 4 g(x)=f(x)-x^4 . Then g ( 1 ) = 1992 g(1)=1992 , g ( 2 ) = 3970 g(2)=3970 , and g ( 3 ) = 5898 g(3)=5898 . Using an interpolating polynomial on this data set, we get g ( x ) = 25 x 2 + 2053 x 36 g(x)=-25x^2+2053x-36 , so f ( x ) = x 4 25 x 2 + 2053 x 36 f(x)=x^4-25x^2+2053x-36 . Now compute f ( 11 ) + f ( 7 ) 4 = 5233 \frac{f(11)+f(-7)}4=\boxed{5233} .

Nice solution! I did the same once I noticed that x 4 {x}^{4} reduces the problem.

Steven Zheng - 6 years, 6 months ago
Aareyan Manzoor
Dec 13, 2014

insert 1,2,3 into the polynomial and we get { a + b + c + d = 1992 8 a + 4 b + 2 c + d = 3970 ( 1 ) 27 a + 9 b + 3 c + d = 5898 \begin {cases} a+b+c+d=1992\\ 8a+4b+2c+d=3970-----(1)\\ 27a+9b+3c+d=5898 \end {cases} subtract 1 from 2, and 2 from 3. we get { 7 a + 3 b + c = 1978 19 a + 5 a + c = 1928 \begin {cases} 7a+3b+c=1978\\ 19a+5a+c=1928 \end {cases} again subtract to get 12 a + 2 b = 50 ( 2 ) 12a+2b =-50---(2) now expand f ( 11 ) + f ( 7 ) = 14641 + 1331 a + 121 b + 11 c + d + 2401 343 a + 49 b 7 c + d f(11)+f(-7)=14641+1331a+121b+11c+d+2401-343a+49b-7c+d f ( 11 ) + f ( 7 ) = 17042 + 988 a + 170 b + 2 ( 2 c + d ) f(11)+f(-7)=17042+988a+170b+2(2c+d) from ---(1), 2 c + a = 3970 8 a 4 a 2c+a=3970-8a-4a , sustitute to get f ( 11 ) + f ( 7 ) = 17042 + 988 a + 170 b + 7940 16 a 8 b f(11)+f(-7) = 17042 +988a+170b+7940-16a-8b f ( 11 ) + f ( 7 ) = 24982 + 81 ( 12 a + 2 b ) f(11)+f(-7) = 24982+81(12a+2b) substitute from---(2) f ( 11 ) + f ( 7 ) = 24982 + 81 × 50 f(11)+f(-7) = 24982+81 \times -50 f ( 11 ) + f ( 7 ) = 20932 f(11)+f(-7)=20932 now 1 4 × 20932 = 5233 \dfrac{1}{4} \times 20932 = \boxed{5233}

Nikola Djuric
Dec 8, 2014

a+b+c+d=1992 (1) , 8a+4b+2c+d=3970 (2) , 27a+9a+3c+d=5898 (3) 1/4(f(11)+f(-7))=1/4(14641+1331a+121b+11c+d+2401-343a+49b-7c+d)= 1/4(17042+988a+170b+4c+2d)= 1/2(8521+494a+85b+2c+d)

Now we want to try to express 494a+85b+2c+d (4) as linear combination of left sides of equalities (1),(2) and (3). (4)=x(1)+y(2)+z(3) which means 27x+8y+z=494 , 9x+4y+z=85 , 3x+2y+z=2 , x+y+z=1 System become x+y+z=1 , 2x+y=1 ,8x+3y=84 , 26x+7y=0 And finally x+y+z=1 , 2x+y=1 , 2x=81 , 12x=486 That means x=40.5 , y=-80 and z=40.5 ,so we can express (4) as combination (1),(2),(3). (4)=40.5(1)-80(2)+40.5(3) so we get
1/2(8521+494a+85b+c)=1/2(8521+40.5(5898+1992)-81x3970) =
1/2(8521+40.5x7890-81x3970)= 1/2(8521+81x3945-317600)=
1/2(8521+319545-317600)= 1/2(8521+1945) =10466/2 =5233 Answer is 5233.


0 pending reports

×

Problem Loading...

Note Loading...

Set Loading...